Graph Theory - Max. Independent set algorithm

Click For Summary
A polynomial time greedy algorithm for computing a maximum independent set in a graph is being sought, but there is confusion regarding the feasibility of this task. Research indicates that finding a maximum independent set is generally not solvable in polynomial time. Participants suggest that the teacher may be asking for an independent set where no additional vertices can be included, which could potentially be solved in O(n^2) time. Clarification on the problem statement is needed to align expectations with the assignment requirements. Understanding the specific definition of "maximum independent set" is crucial for developing the appropriate algorithm.
oneamp
Messages
219
Reaction score
0
Graph Theory -- Max. Independent set algorithm

Homework Statement



Design a polynomial time greedy algorithm to compute a maximum independent set for a graph. Explain the algorithm and compute T_w(n).


Homework Equations





The Attempt at a Solution



My terse and informal Internet research indicates that a maximum independent set cannot be calculated in polynomial time. For example:
http://en.wikipedia.org/wiki/Independent_set_(graph_theory)#Finding_maximum_independent_sets

What could my teacher possibly want? Do I need to design an algorithm that nobody else to date has done? I guess not. I think I misunderstand the question. Can someone help me understand what he wants?

Thank you
 
Physics news on Phys.org
Maybe "maximum independent set" is not meant in that way, and you are supposed to find an algorithm that finds an independent set where no other vertices can be added (should be easy in n^2). Just a guess, I don't know how the problem statement is meant.
 
Question: A clock's minute hand has length 4 and its hour hand has length 3. What is the distance between the tips at the moment when it is increasing most rapidly?(Putnam Exam Question) Answer: Making assumption that both the hands moves at constant angular velocities, the answer is ## \sqrt{7} .## But don't you think this assumption is somewhat doubtful and wrong?

Similar threads

  • · Replies 1 ·
Replies
1
Views
2K
  • · Replies 10 ·
Replies
10
Views
3K
  • · Replies 4 ·
Replies
4
Views
2K
  • · Replies 2 ·
Replies
2
Views
2K
  • · Replies 7 ·
Replies
7
Views
3K
  • · Replies 1 ·
Replies
1
Views
2K
  • · Replies 1 ·
Replies
1
Views
2K
  • · Replies 10 ·
Replies
10
Views
2K
Replies
2
Views
2K
  • · Replies 9 ·
Replies
9
Views
4K